Selectivo de IMO 2011 - Problema 5

Problemas que aparecen en el Archivo de Enunciados.
Avatar de Usuario
Matías V5

Colaborador-Varias OFO - Jurado-OFO 2015 OFO - Jurado-OFO 2016 FOFO 6 años - Jurado-FOFO 6 años OFO - Jurado-OFO 2017
OFO - Jurado-OFO 2018 OFO - Jurado-OFO 2020 OFO - Jurado-OFO 2021
Mensajes: 1115
Registrado: Dom 17 Oct, 2010 4:44 pm
Medallas: 8
Nivel: Exolímpico

Selectivo de IMO 2011 - Problema 5

Mensaje sin leer por Matías V5 »

En un torneo de tenis participan por lo menos [math] jugadores. Cada uno juega exactamente un partido contra cada uno de los demás. Además, cada jugador gana al menos uno de los partidos que juega. (En tenis no hay empates.)
Demostrar que en el torneo hay (al menos) tres jugadores [math] tales que [math] le gana a [math], [math] le gana a [math] y [math] le gana a [math].
We gave you a start so you'd know what to do
You've seen how it works, now it's over to you (...)
For there's so much more to explore!

Numberblocks - https://www.youtube.com/watch?v=KzTR72_srTU
Avatar de Usuario
Vladislao

Colaborador-Varias OFO - Jurado-OFO 2015 OFO - Jurado-OFO 2016 FOFO 6 años - Jurado-FOFO 6 años OFO - Jurado-OFO 2017
FOFO Pascua 2017 - Jurado-FOFO Pascua 2017
Mensajes: 808
Registrado: Mar 28 Dic, 2010 3:26 pm
Medallas: 6
Nivel: Exolímpico
Ubicación: Córdoba

Re: Selectivo de IMO 2011 - Problema 5

Mensaje sin leer por Vladislao »

Spoiler: mostrar
Sea [math] la cantidad de jugadores [math]. Sea [math] la cantidad de partidos que ganó el que menos partidos ganó (o uno de los que menos partidos ganó). Sea [math] un jugador que ganó [math] partidos.

Si [math], [math] ganó un solo partido. Pongamos que [math] sea el único que perdió contra [math]. Entonces hay [math] jugadores que le ganaron a [math]. Si [math] le gana a alguno de esos [math] jugadores, estamos listos. Supongamos, por el contrario, que [math] pierde contra los [math] que le ganaron a [math], como [math] perdió contra [math] entonces se tiene que [math] perdió contra todos. Absurdo, pues cada jugador ganó, al menos, un partido.

Por inducción en [math], supongamos que es cierto hasta cierto valor, digamos [math]. Veamos qué pasa con [math].

Sea, nuevamente, [math] el jugador que ganó [math] partidos. Entonces existen [math] jugadores que le ganaron a [math] y [math] que perdieron contra [math]. Sean [math] los que perdieron contra [math]. Si algún [math] le ganó a alguno de los [math] jugadores que le ganaron a [math], estamos listos. Supongamos que no, entonces los [math] jugadores que perdieron contra [math], se ganaron entre sí. Es claro que cada uno de estos jugadores ganó al menos [math] partidos. Pero tiene [math] rivales a los cuales les puede ganar. Entonces, algún [math] le ganó a alguno de los [math] jugadores que le ganaron a [math], y por lo tanto existe una terna como la que propone el enunciado.
Sea [math] Para todo entero positivo [math] se cumple que [math] es un número primo.
cogo

Colaborador-Varias OFO - Jurado-OFO 2015
Mensajes: 31
Registrado: Vie 29 Oct, 2010 9:24 pm
Medallas: 2

Re: Selectivo de IMO 2011 - Problema 5

Mensaje sin leer por cogo »

Spoiler: mostrar
Primero que nada es fácil ver que para n=3 se encuentra una terna como la del enunciado.
Supongamos ahora como hipótesis inductiva que para k jugadores hay una terna como la del enunciado.
Vamos a demostrar que entonces para k+1 jugadores encontramos una terna como la del enunciado. Supongamos a modo de contradicción que no hay una terna como la del enunciado para k+1 jugadores.
En efecto, veamos que cada jugador debe perder al menos un partido pues, de lo contrario, si un jugador gana todos los partidos tendremos k jugadores en las mismas condiciones que en un torneo con k jugadores por lo que, por la hipótesis inductiva, encontraremos la terna buscada. Luego, cada jugador debe perder al menos un partido.
Sea A el jugador que más partidos ganó (o al menos uno de los que ganó mas partidos).
Como cada jugador juega k partidos y en total se juegan (k+1)*k/2 partidos A debe ganar al menos k/2 partidos.
Sean b1, b2, ..., bt con t >= k/2 los jugadores que perdieron con A y sean c1, c2, ..., cq con q <= k/2 los jugadores que le ganaron a A. Veamos que cada uno de los ci debe ganarle a cada uno de los bj pues de lo contrario tendríamos la terna A--G-- bj --G-- ci --G-- A.
Pero entonces como cada uno de los ci le gana a cada uno de los bj y como además cada uno de los ci le gana a A cada uno de los ci ganará más partidos que A por lo que ésto se contradice con nuestra definición de A como el jugador que más partidos gana en el torneo.
Avatar de Usuario
amcandio

Colaborador-Varias
Mensajes: 313
Registrado: Sab 16 Oct, 2010 12:50 pm
Medallas: 1
Nivel: Exolímpico
Ubicación: Posadas, Misiones
Contactar:

Re: Selectivo de IMO 2011 - Problema 5

Mensaje sin leer por amcandio »

Spoiler: mostrar
Supongmos que para todo numero m entre 3 y n, se cumple que para cada torneo de tenis todos contra todos de m personas, tales que uno le gano a al menos uno de los otros, se cumple con la existencia de [math], [math], y [math]. (1)

Ahora supongamos que con n+1 no existen [math], [math], y [math], entonces tomamos a un participante, digamos X y dividimos a el resto de los participantes en Y, que contiene a todos los que le ganaron a X, y en Z, que contiene a todos los que perdieron contra X. Ninguno de los de Z le pueden ganar a los de Y, entonces tiene mas de 2 personas, y se cumple que cada uno de los de Z le gana a por lo menos uno de los de Z, jugando una vez con cada uno. Como Z tiene entre n y 3 elementos sigue por (1) que existen [math], [math], y [math]. ABSURDO.
Luego existen [math], [math], y [math], para n+1.

Con 3 participantes es trivial que existen los tres participantes, luego por induccion, se puede para todo n.
"Prillo es el Lanata de la trigonometria"
sebach

Colaborador-Varias OFO - Medalla de Bronce-OFO 2017 OFO - Medalla de Bronce-OFO 2018 OFO - Medalla de Bronce-OFO 2020 OFO - Medalla de Plata-OFO 2021
OFO - Medalla de Plata-OFO 2022 OFO - Medalla de Plata-OFO 2023 OFO - Medalla de Oro-OFO 2024
Mensajes: 202
Registrado: Dom 06 Mar, 2011 11:49 am
Medallas: 8
Nivel: Exolímpico

Re: Selectivo de IMO 2011 - Problema 5

Mensaje sin leer por sebach »

Una simple:
Spoiler: mostrar
DIgamos que el jugador que menos partidos gana, A, gana X partidos. Luego, agarramos a una de estas X personas, ya que X>0 siempre, y decimos que esta persona debe ganarle a por lo menos X personas, ya que la otra persona con X ganados era la que menos partidos gano. Como debe ganarle a X personas al menos, les puede ganar a todos los que le ganó A, pero ahi hay X-1, y entonces debe ganarle a alguna de las otras personas. Pero todo el resto de las personas le gano a A. Entonces tenemos el trio que buscabamos.
ACLARACION: Si no hubiera otra persona aparte de A y de estas X personas, entonces A les gano a todos, pero era el que menos partidos habia ganado. Entonces sí o sí hay por lo menos otro jugador.
2  
Avatar de Usuario
amcandio

Colaborador-Varias
Mensajes: 313
Registrado: Sab 16 Oct, 2010 12:50 pm
Medallas: 1
Nivel: Exolímpico
Ubicación: Posadas, Misiones
Contactar:

Re: Selectivo de IMO 2011 - Problema 5

Mensaje sin leer por amcandio »

me gusta, la mia es una bosta, xD pero fue la primera que se me ocurrio
"Prillo es el Lanata de la trigonometria"
Alejo

OFO - Medalla de Plata-OFO 2015 OFO - Jurado-OFO 2016 OFO - Jurado-OFO 2017
Mensajes: 76
Registrado: Dom 01 Abr, 2012 4:14 pm
Medallas: 3
Nivel: Exolímpico

Re: Selectivo de IMO 2011 - Problema 5

Mensaje sin leer por Alejo »

Spoiler: mostrar
Decimos que para dos jugadores A y B, A>B si A le ganó a B. Supongamos que no se cumple lo que hay que demostrar. Entonces si tomamos tres jugadores A, B y C tales que A>B y B>C, entonces no se cumple que C>A, luego A>C. Con esto se ve que la relación > es una relación de orden estricta y total:
a) Para todo A no se cumple que A>A: verdadero porque A no juega consigo mismo.
b) Si A>B entonces no se cumple B>A: verdadero porque los jugadores solo juegan una vez entre sí y no pueden ganar y perder al mismo tiempo.
c) Si A>B y B>C entonces A>C: es lo que probamos en el párrafo anterior.
d) Para A y B distintos se cumple A>B o B>A: verdadero porque todos los jugadores juegan entre sí y no hay empate.
Entonces el conjunto de todos los jugadores está totalmente ordenado. Como es finito, tiene un mínimo; ese mínimo, por definición es menor a todos los demás, luego no le ganó a nadie, absurdo. Queda demostrado.
Avatar de Usuario
julianferres_

OFO - Medalla de Bronce-OFO 2015 OFO - Medalla de Plata-OFO 2016 OFO - Medalla de Plata-OFO 2017 OFO - Mención-OFO 2021
Mensajes: 388
Registrado: Sab 17 Sep, 2011 8:01 pm
Medallas: 4
Nivel: Exolímpico
Ubicación: Villa Ramallo, Buenos Aires

Re: Selectivo de IMO 2011 - Problema 5

Mensaje sin leer por julianferres_ »

Selectivo IMO 2011 P5.pdf
No tienes los permisos requeridos para ver los archivos adjuntos a este mensaje.
Avatar de Usuario
Violeta

OFO - Mención-OFO 2017 FOFO 7 años - Medalla Especial-FOFO 7 años OFO - Medalla de Bronce-OFO 2018 FOFO 8 años - Mención Especial-FOFO 8 años OFO - Medalla de Plata-OFO 2019
Mensajes: 405
Registrado: Sab 04 Jun, 2016 11:50 pm
Medallas: 5
Ubicación: Puerto Rico

Re: Selectivo de IMO 2011 - Problema 5

Mensaje sin leer por Violeta »

Spoiler: mostrar
Diremos que los jugadores $a_1,a_2, \ldots , a_k$ están en un ciclo de tamaño $k$ si $a_i$ le ganó a $a_{i+1}$, tomando los índices $\pmod{k}$.

Primero, como cada jugador le ganó a por lo menos un jugador, y no hay empates, definitivamente hay un ciclo entre los jugadores. Digamos que de todos los posibles ciclos, el más pequeño tiene tamaño $k \neq 3$, para probarlo por contradicción.

Luego, $a_1$ debe haberle ganado a $a_3$, porque sino habría un ciclo de tamaño tres: $a_1,a_2,a_3$. Pero entonces $a_1, a_3, \ldots , a_k$ es un ciclo de menor tamaño. Absurdo porque habíamos dicho que el de menor tamaño tiene tamaño $k\neq 3$. Entonces, el ciclo más pequeño tiene tamaño $3$ y estamos.
Para todo [math], existen [math] primos en sucesión aritmética.
Avatar de Usuario
Gianni De Rico

FOFO 7 años - Mención Especial-FOFO 7 años OFO - Medalla de Oro-OFO 2019 FOFO 9 años - Jurado-FOFO 9 años COFFEE - Jurado-COFFEE Matías Saucedo OFO - Jurado-OFO 2020
FOFO Pascua 2020 - Jurado-FOFO Pascua 2020 COFFEE - Jurado-COFFEE Carolina González COFFEE - Jurado-COFFEE Ariel Zylber COFFEE - Jurado-COFFEE Iván Sadofschi FOFO 10 años - Jurado-FOFO 10 años
OFO - Jurado-OFO 2021 FOFO 11 años - Jurado-FOFO 11 años OFO - Jurado-OFO 2022 FOFO Pascua 2022 - Jurado-FOFO Pascua 2022 FOFO 12 años - Jurado-FOFO 12 años
OFO - Jurado-OFO 2023 FOFO 13 años - Jurado-FOFO 13 años OFO - Jurado-OFO 2024 FOFO Pascua 2024 - Jurado-FOFO Pascua 2024
Mensajes: 2222
Registrado: Vie 16 Sep, 2016 6:58 pm
Medallas: 19
Nivel: Exolímpico
Ubicación: Rosario
Contactar:

Re: Selectivo de IMO 2011 - Problema 5

Mensaje sin leer por Gianni De Rico »

Solución:
Spoiler: mostrar
Sea $n$ la cantidad de jugadores. Consideramos el grafo dirigido $G$ en el que los vértices son los jugadores, y para cada par de vértices $u,v$, existe la arista $(u,v)$ si y sólo si $u$ le ganó a $v$. Como cada jugador ganó al menos un partido, el grado de salida $d^+(v)$ de cada vértice $v$ cumple que $d^+(v)\geq 1$, y como cada jugador juega exactamente un partido contra cada uno de los demás, $G$ es simple y completo, y al ser dirigido, es un torneo. Luego, tiene un camino hamiltoniano dirigido, sea $v_1,\ldots ,v_n$ dicho camino, como $d^+(v_n)\geq 1$ y existe $(v_{n-1},v_n)$, tenemos que existe $(v_n,v_i)$ para algún $1\leq i\leq n-2$, es decir, $v_i,\ldots ,v_n$ es un ciclo, y como $G$ es un torneo, esto implica que contiene un ciclo de tamaño $3$. Luego, existen tres vértices $A,B,C$ tales que $(A,B),(B,C),(C,A)$ son aristas de $G$, y por la definición de $G$, tenemos que exiten $3$ jugadores $A,B,C$ tales que $A$ le gana a $B$, $B$ le gana a $C$ y $C$ le gana a $A$.
♪♫ do re mi función lineal ♪♫
Responder